You are on page 1of 17

A Model for Collisions Between

Two Solid Objects


One way mechanical energy is lost during collision between two solid objects is by way of
acoustic waves propagating inside both objects. Even though the real situation is quite
complicated, in this problem it is modeled in a simple way. First treat each solid rod as a
spring with unstretched length Ll and Lr . Spring constant times the length of the spring
are then Kl and Kr , respectively. Mass density (mass per unit length) of the spring are
given by l and r . Indexes l and r represent the left and right springs.
The left spring is moving with velocity +v0 /2, while the right spring is moving in the
opposite direction with velocity v0 /2. Each spring is initially relaxed. At t = 0, the
springs collide at x = 0. The displacement of each point on the springs are described by a
function y(x, t) so that x + y(x, t) represents the position of said point at time t; this point
was at x when t = 0.
1. Derive the wave equation on the springs and write down the speed of the wave.
The general solution to the wave equation is given by y(x, t) = (ct x) + (ct + x), where
c is the speed of the wave propagation. The form of and are determined from the
boundary conditions.
2. Write down the boundary conditions at x = 0, x = Ll and x = Lr .
3. Write down the function of y(x, t) before the collision (t 0), i.e. y0,l (x, t) and
y0,r (x, t).
At t = 0, an acoustic wave starts to propagate in both springs away from the collision point
x = 0. The dynamics of the system is analyzed using the space-time diagram as shown in
Fig. 1. The horizontal axis represents time, and the vertical axis represents the position of
points on the spring. Each line in the diagram represents an acoustic wave front that
emerges each time a wave front arrives at the border.
For example, the line AB represents a wave front emerging from the collision at point A
(x = 0). Waves propagating in the left and right springs are given by functions fl (cl t + x)
and fr (cr t x), where cl and cr are the speed of the wave propagation in left and right
spring, respectively. The space-time diagram indicates that Ll /cl > Lr /cr in this problem.
As the wave front of fr (cr t x) arrives at point B, a new reflected wave, gr (cr t + x),
emerges. The same event also occurs in the left spring at point C.
Now back in the right spring, when the wave front of gr (cr t + x) arrives at the end of the
spring (x = 0, at point D in the diagram), a new reflected wave hr (cr t x) and a new
transmitted wave hl (cl t + x) are generated. These phenomena always occur when a wave
front arrives at the border; a new reflected wave or new reflected and transmitted waves
are generated.
4. Write down the wave function y(x, t) in the region I, II, III, IV, V, VI and VII in terms
of y0 , fr , fl , gr , hr and hl .

Page 1 of 2

Figure 1: Space-time diagram


5. Using the boundary condition(s), determine the form of fr (cr t x) and fl (cl t + x) in
terms of the springs properties and initial velocity.
6. Determine the velocity of the contact point (x = 0) immediately after the collision.
7. Using the boundary condition(s), determine the form of gr (cr t + x) in terms of the
springs properties and initial velocities.
Now consider a case where both springs are identical except in its length. In this case,
l = r = , Kl = Kr = K. Take Lr < Ll .
8. Determine y(x, t) in region III and IV. Draw a graph for y(x) at t = 0.4 Lc . For drawing
the graph, you may use Lr = 0.6L, Ll = L and v0 = 0.5c.
9. Determine y(x, t) in region V. Draw a graph for y(x) at t = 0.8 Lc , use the same Lr , Ll
and v0 as in the previous question.
10. When will the two springs separate? Draw a graph for y(x), use the same Lr , Ll and v0
as in the previous question.
11. Calculate the coefficient of restitution e between the springs.
12. Calculate the ratio of the translational kinetic energy of the springs after the collision
to the kinetic energy before the collision.
Page 2 of 2

Spring and Mass


Problem
1. A mass M moves toward a semi infinite spring with initial velocity v0 , as shown in
Fig. 1. The spring has mass per unit length and spring constant times the spring
length K kL. The mass and the spring collide at x = 0 and t = 0. Write down the
velocity of the mass M after the collision as a function of time, and also write down the
velocity of the mass M as a function of position.

M
v0

Figure 1:
2. For this part another mass m is placed at the other end of the spring. After the initial
wave front from the collision of mass M with the spring reaches this mass, how long
would it take for this mass to leave the spring? Also calculate the velocity of mass m
when it leaves the spring. Assume the waves in the spring travel faster than the initial
velocity of mass M but the spring is long enough so that when the mass m leaves the
spring, the reflected waves from m have not yet returned to M .

Page 1 of 1

Sliding Block
A rectangular block of width 2b, length 2a, and mass M rests on a rough surface which has
a coefficient of kinetic friction . At some time, the block is given a sharp kick, such that it
suddenly attains a horizontal velocity v0 . Under certain circumstances the rear end of the

Figure 1: The block, after given its initial velocity.


block will begin to lift and the block will subsequently rotate about its front lower edge,
which will remain in contact with the surface.
1. Derive the equation of rotational motion of the block in terms of , a, b, , and g.
2. Find the physical condition, namely the range of , that allows this to happen.
The next question assume this condition is fulfilled, and concerns the subsequent motion of
the block.
3. Consider a final state in which the block is at rest in the position shown in Fig. 2,
where its center of mass has undergone a total horizontal displacement x. Is such a

Figure 2: A presumed final position of the sliding block.


position possible? If yes, calculate the initial velocity required to achieve it for the
following values: a = 0.8 m, b = 1.0 m, = 0.9, x = 1.65 m, max = 1.27 s1 .
Note: Knowing a, b and the initial velocity can be solved numerically.

Dielectric Slab
Waveguide
1

Total Internal Reflection

The electric field of a polarized monochromatic plane wave can be generally represented as
E(r, t) = E exp i(k.r t), where E is the amplitude of the wave, k the wavenumber, and
the frequency. Suppose that a monochromatic plane wave with frequency travels in
the medium of refractive index n1 , and is incident on the boundary of another medium of
refractive index n2 . The incoming wave forms an angle i with respect to the normal of
the boundary. Throughout this problem, we only consider transverse electric (TE) polarized
wave where the electric field is perpendicular to the plane of incidence and all media are
non-magnetic.

1. In the case of n1 > n2 , there exists a critical angle c where the incoming wave will be
totally reflected for i > c (total internal reflection or TIR). The phase of the reflected
wave lags by with respect to the incident wave. Derive and state it in terms of n1 ,
n2 , and i .
2. Using the necessary boundary conditions, derive the reflectance R for the case of TIR.
Show that the wave is perfectly reflected for all i > c .

Constructive Phase Matching

The most simple dielectric waveguide is a planar slab with thickness d and refractive index
n1 located in a homogeneous background medium with refractive index n2 (n2 < n1 ). In the
case of TIR, the slab can be used to guide waves without loss, with the additional condition
that the waves interfere constructively. In other words, the wavefronts should be preserved
as the waves travel inside the waveguide. The wavenumbers for the vacuum, medium n1 ,
and medium n2 are taken to be k0 , k1 , and k2 , respectively.

1. Find the necessary condition for the constructive phase matching.


2. The wave can only be guided without loss for certain values of . Show that in these
cases, must satisfy the equations:
k1 d cos = m;

m = 0, 1, 2, 3, ....

Verify that the equations above can also be written as:


q

k0 d
2
2
u +v =
n21 n22 ,
2
u tan u = v
or
p
with u = k21 d cos and v = d2 k12 sin2 k22 .

u cot u = v,

(1)

(2)
(3)

Maxwells Equations

The Maxwell wave equation for the electric field in a dielectric medium of relative permittivity
is
 2


2
2
2 E(r, t)
+
+
.
(4)
E(r,
t)
=

0
0
x2 y 2 z 2
t2

In the case of the slab waveguide shown in the figure above, = n21 for 0 < z < d, and
= n22 for z < 0 or z > d. Taking the system coordinates such that the wave travels in the
xz-plane, the electric field can be generally written as
E(r, t) = E(x, z, t) = E(z) exp i(x t),

(5)

where is the effective propagation constant along the waveguide due to the translational
symmetry of the structure in the x-direction. In the case of waveguiding the TE polarized
wave (E(z) = E(z)
y), E(r, t) should be simple harmonic inside the slab and decay
exponentially outside.
1. What is the relation of to k1 and ?
2. From the boundary conditions at z = 0 and z = d, derive from the Maxwell equations
the condition for waveguiding as found in Part 2.
3. Sketch the functions in eqs. (1)-(2) in (u, v) coordinates. Determine the necessary
condition for only one solution of to exist.

Page 2

Mode Solutions

The waveguide mode solutions are solutions of where waveguiding occurs inside the slab.
The solution for m = 0 (see Part 2) is commonly called the fundamental mode (the lowest
mode or the first mode), the m = 1 mode is called as the second mode, and so on.
1. Sketch the functions in eqs. (2)-(3) in (u, v) coordinates. Determine the necessary
condition for only one mode solution to exist.
2. Show that the maximum number of modes supported by the dielectric slab is


q
k0 d
2
2
M=
n1 n2 ,

(6)

where the de symbol denotes the ceiling function for which the expression inside is
increased to the nearest integer.
3. Verify that the number of mode solutions is incremented by one for every increase of
frequency:
c
= p 2
.
(7)
d n1 n22
4. From eq.1, show that the group velocity (d/d) of each supported mode solution is
vg =

d tan +
n1 d
c cos

(8)

5. Show that the maximum time disparity for different modes in the dielectric slab
waveguide to travel a distance L is
=

L
(n1 n2 ).
c

6. For n1 = 1.7, n2 = 1.5, = 800 nm (in vacuum), and d = 1 m, find all the mode
solutions for (with > c ). Plot the electric field E(z) for these solutions.

Page 3

(9)

Liquid Air
A mixture of oxygen and nitrogen gas is stored in a closed container equipped with a
piston on one end at a temperature of T = 77.4 K. The total amount of the gas mixture
is 1.1 mole and its initial pressure is 0.5 atm. With the help of the piston the gas mixture
is slowly compressed at constant temperature.
Using plausible assumptions, plot the pressure of the system as a function of its volume
until one tenth of the initial volume, if the ratio of the number of moles of oxygen to the
number of moles of nitrogen is
nO2
1
= .
nN2
9
nO2
2
= .
b)
nN 2
9
nO2
1
c)
= .
nN 2
4
a)

Find the pressure and volume at distinctive points of these isothermal curves.
You can use the following data:
Boiling point of liquid nitrogen at 1 atmosphere: 77.4 K
Boiling point of liquid oxygen at 1 atmosphere: 90.2 K
Heat of vaporization of oxygen: 213 J/g.

Lagrange Points
Stability
In a system that rotates with the Earth around the Sun, there are five equilibrium points
(where the sum of the forces is zero). These 5 points are known as Lagrange Points (named
after Joseph Lagrange, the first person to study this three-body system). Exact analysis of
this system is very complicated and chaotic. In the following problem, the mass of the two
bodies (M1 and M2 ) are taken to be much larger than that of the third body (m). The
distance between M1 and M2 is taken to be R.

rm 1

rm 2

M2
M1

r1

r2
R

1. Basic equations of the system


(a) Write down the vector of the total gravitational forces Fg on m.
(b) By assuming M1 , M2 >> m, determine the angular velocity of the M1 and M2
system ().
(c) In a frame that rotates with the system, there are fictitious forces on m. Write
down the vector of the total forces on this mass (F ) in this frame.
(d) Choose a coordinate system where the three masses are in the xy-plane and the
angular velocity is in the positive z-axis. The center of the coordinate is set at
the center of mass of M1 and M2 on the x-axis. Write the position of m as
r = x(t)i + y(t)j. In the rotating frame where m is at rest, write down the total
1
2
and = M1M+M
.
forces on m in the x- and y-axis using parameter = M1M+M
2
2
2. Identifying Lagrange Points
There are 5 points with zero net forces in this rotating system. Three of them (call
them L1 , L2 and L3 ) lie on the line connecting M1 and M2 (the x-axis) and the other
two (call them L4 and L5 ) lie on the xy-plane on symmetric positions above and below
the x-axis; that is, y4 = y5 .

(a) First consider the case of finding the position of L1 , L2 and L3 . Use x = ( )R,
with the distance of m from M1 in units of R. Write down the equation of force
that must be satisfied to identify these points. Express this equation in terms of
and .
(b) The equation above gives rise to three cases (each for L1 , L2 and L3 ) to consider,
< a, a < < b and b < . Determine the values of a and b.
From here on, we will also assume that is small (in the Earth-Sun system, is
3.0 106 ). Use only the lowest order non-zero term in , ignore all higher order terms
in . The following three questions will help you determine the three Lagrange points
on the x-axis.
(c) For the first case, < a, write = 1 + 1 with 1 a small positive number that
depends on . This value of will determine the position of the first Lagrange
point at x = R(1 + 1 ). Determine 1 as a function of .
(d) For the second case, a < < b, write = 1 2 with 2 a small positive number
that depends on . This value of will determine the position of the second
Lagrange point at x = R(1 2 ). Determine 2 as a function of .
(e) For the third case, b < , write = 1 + 3 with 3 a small positive number that
depends on . This value of will determine the position of the third Lagrange
point at x = R(1 + 3 ). Determine 3 as a function of .
Determining the fourth and fifth Lagrange points requires a more complicated method.
First decompose the gravitational force on m into components parallel and
perpendicular to the vector r.
(f) Find the unit vector parallel to the vector r,
ek . Find also the unit vector
perpendicular to the vector r on the xy-plane,
e .
k

(g) Find the component of the force on m parallel to the vector r, F , and find the
component perpendicular to the vector r, F .
(h) Specify the condition that must be satisfied by the force component perpendicular
to the vector r in order that mass m be in equilibrium. With this condition,
determine the relation between rm1 and rm2 .
(i) Specify the condition that must be satisfied by the force component parallel to the
vector r in order that mass m be in equilibrium. With this equation, determine
the relation between rm1 and R.
(j) Now determine the position of the fourth Lagrange point (x4 , y4 ) and the fifth
Lagrange point (x5 , y5 ).
3. Lagrange Point Stability
To test the stability of these Lagrange points, small perturbation are given to the mass
m around its equilibrium points. Because the forces in this system depend on the
Page 2

position (x, y) and the velocity (vx , vy ) of the mass m, the restoring forces must be
calculated for variations in position and velocity. Expand the total force as follows:
Fx
Fx
Fx
Fx
x +
y +
vx +
vy
x
y
vx
vy
Fy
Fy
Fy
Fy
Fy (x0 + x, y0 + y, vx,0 + vx , vy,0 + vy ) =
x +
y +
vx +
vy .
x
y
vx
vy

Fx (x0 + x, y0 + y, vx,0 + vx , vy,0 + vy ) =

This force has taken into account the contribution of the velocity of the mass m. All
the partial derivatives are evaluated at the point (x0 , y0 , vx,0 , vy,0 ).
(a) Write down the general form for
x
, 1 Fx , 1 Fy ,
(b) Calculate m1 F
vx m vy m vx
the five Lagrange points.

1 Fx 1 Fx 1 Fy 1 Fy
, m y , m x , m y .
m x

1 Fy
.
m vy

Show that

Fy
x

Fx
.
y

These four coefficients should not change for

These eight coefficients should act as a restoring constant (analog to the spring
constant). Now we are ready to check the stability of the five Lagrange points.
Consider only the lowest order term in , ignore all higher order terms.
(c) The first Lagrange Point
x
= c1 2 . Determine c1 .
i. Show that m1 F
x
ii. Show that
iii. Show that

Fy
x
= F
= 0.
x
y
F
y
1
= c2 2 .
m y
t

Determine c2 .

iv. By substituting x = Ae and y = Bet , with A and B nonzero, determine


as a function of and only.
v. There are four solutions to . Write down the condition that these solutions
must satisfy in order that the first Lagrange point is stable and then
determine the stability of this point.
vi. For the Earth-Sun system is 3.0 106 and is 2/year. If this point is
stable, determine its period of oscillation (in days), if not, determine its time
constant 1/ (in days also).
(d) The second Lagrange Point
x
= c3 2 . Determine c3 .
i. Show that m1 F
x
ii. Show that
iii. Show that

Fy
x
= F
= 0.
x
y
1 Fy
= c4 2 . Determine
m y
t

c4 .

iv. By substituting x = Ae and y = Bet , with A and B nonzero, determine


as a function of and only.
v. There are four solutions to . Write down the condition that these solutions
must satisfy in order that the second Lagrange point is stable and then
determine the stability of this point.
Page 3

vi. For the Earth-Sun system: if this point is stable, determine its period of
oscillation (in days), if not, determine its time constant 1/ (in days also).
The third Lagrange point is similar to the second Lagrange point hence it need not be
considered.
(e) The fourth Lagrange Point
x
= c5 2 . Determine c5 .
i. Show that m1 F
x
ii. Show that
iii. Show that

1 Fy
m x
1 Fy
m y

1 Fx
m y

= (c6 + c7 )2 . Determine c6 and c7 .

= c8 2 . Determine c8 .

iv. By substituting x = Aet and y = Bet , with A and B nonzero, determine


as a function of and only.
v. Define M1 /M2 = . Find the range of value of for the fourth Lagrange point
to be stable.
The fifth Lagrange point has the same behavior as the fourth Lagrange point, hence it
need not be considered.

Page 4

Helical Rope
A device is built by placing two metal strips, each of mass m, on a large, frictionless
cylinder. Two identical massless elastic ropes each with spring constant k and obeying
Hookes Law are used to connect the metal strips such that the two ropes are initially at
their natural length x0 and parallel to each other. The contact points of each rope on the
same strip are diametrically opposite to each other, and the whole device can be seen on
Figure 1. Strip A is bolted to the cylinder, while strip B is free to move along and rotate
about the cylinders axis.

Figure 1: The device in its initial configuration. The screw can be used to prevent rotation
of strip B.
1. The cylinder is now oriented such that its axis is vertical under a constant
gravitational field g, and strip A is above strip B. Strip B is now rotated N times while
its distance from strip A is maintained at x0 . After this, strip B is prevented from
rotating by a screw, as shown in Figure 1.
(a) Find an equation that, given numerical values for the initial conditions, would
allow you to calculate x1 , the new equilibrium position.
(b) Under certain conditions the metal strip will undergo simple harmonic oscillation.
Calculate the frequency of oscillation for small x, in terms of k, r, N , x0 , and x1 .
2. The cylinder is now oriented horizontally, the ropes are returned to their initial
position, and strip B is prevented from rotating by means of the screw.
(a) A horizontal stretching force F is now applied to strip B. If the force is increased
very gradually, the ropes break when the force reaches a value F0 . What, then, is
the minimum amount of constant force required to break the ropes?
(b) If strip B is rotated N times before the screw is put in place keeping the total
length of the ropes x0 , calculate the minimum horizontal force required to break
the ropes if said force is:
i. increased very gradually.
ii. kept constant.

3. The system is returned to its initial horizontal configuration. All constraints on strip B
are now removed, and the strip is rotated through an angle 0 while keeping the
distance between the two metal strip x0 , and then released (initially x(0)

= 0, and

(0)
= 0).
(a) Find the equation of motion of strip B!
(b) Solve the equation of motion for x (t) and (t)!
(c) Find the maximum velocity and maximum angular velocity, and also the time T
required for strip B to reach strip A!

Page 2

Magnetic Dipole
Oscillation
A magnetic dipole with magnetic moment m1 is placed at the coordinate origin parallel to the
x-axis.
1. Determine the resulting magnetic field in all space.
2. Another dipole is placed at a distance r from the origin at an angle to the x-axis. The
magnetic moment of the second dipole, m2 , forms an angle to the x-axis. The whole set-up
can be seen in Fig. 1. Determine the torque on the second dipole.
3. Determine the interaction energy between the two dipoles.
4. Determine the force on the second dipole.
5. The second dipole is tied to the first dipole via a massless string such that the distance
between the two is fixed at R. While the orientation of the first dipole at the coordinate origin
is fixed, the orientation of the second dipole may change. It is also allowed to move freely in
the xy-plane around the first dipole. Write down the equation of motion of the second dipole.
The mass and moment of inertia of the second dipole are taken to be m and I respectively.
6. Initially the second dipole is at rest on the x-axis, with the magnetic moment forming an
angle 0 to the x-axis (0 << 1). At t = 0, the second dipole is released and allowed to move
freely. Write down the equation of motion of the second dipole assuming and are small.
7. The system undergoes simple harmonic oscillation. You are asked to determine the normal
mode frequencies of oscillation. The system is in a normal mode when the oscillating variables
are in phase and can be written as follows: = 0 cos (t + ) and = 0 cos (t + ). There
are two possible values of (denoted by 1 and 2 ). Determine 1 and 2 .
8. For each normal mode, determine the ratio of the amplitude of to , c1 = 1 /1 and
c2 = 2 /2 .
9. The equation of motion of the system can be expressed as follows:
= 1 cos (1 t + 1 ) + 2 cos (2 t + 2 ) ,
= c1 1 cos (1 t + 1 ) + c2 2 cos (2 t + 2 ) .
Using the initial conditions, determine the values of 1 , 1 , 2 , and 2 .

Figure 1:

Cylinder Collision
A hollow cylinder with mass M and radius R is at rest on a horizontal plane. In the interior
of this cylinder, there is a solid disk with mass m and radius r. Initially, the center of the
disk is at a distance l from the center of the cylinder and moves with velocity v y as shown
in Fig. 1. Unless otherwise specified, all collisions are elastic and frictions can be ignored.

Figure 1:

1. Determine the velocity (the x and the y components of the velocity) of the disk and
the cylinder immediately after the first collision. Write your answer in terms of m, M ,
v and .
2. Determine the velocity (the x and the y components of the velocity) of the disk and
the cylinder immediately after the second collision. Write your answer in terms of m,
M , v and .
3. If initially the disk is placed at l = (R r)/2, determine the velocity of the disk and
the cylinder immediately after the n-th collision.
4. What is the condition for l such that immediately after the n-th collision m moves with
velocity v y and M is at rest? Determine the distance between two successive positions
of the center of M when it is at rest.
5. For this part, the friction between the the disk and the cylinder cannot be ignored. As
in part (a), initially the cylinder is at rest, while the center of the disk is at a distance
l < (R r) from the center of the cylinder and moves with velocity v y as shown in
Fig. 1. If during the collision process the point of contact does not slide, determine the
angular velocity of the disk and the cylinder immediately after the first collision.

Charged Discs
Two thin metal discs of radius 5 cm each are suspended by electrically insulating threads
such that the discs are parallel (see Fig. 1a) and close to each other (for example their
distance could be 2 mm).
1. Calculate the force between the two discs if they are charged with small charges +q
and q respectively. As q is small, the displacement of the discs and the possibility of
electric discharge can be neglected.
2. Now consider only one disc; calculate the surface charge distribution on a metal disc of
radius R having total charge +q. (This charge distribution might be useful to answer
the next question.)
After this, the two original discs are each charged +q. A third metal disc of radius R > 5
cm is carefully inserted between the two discs; the third disc is neutral and is suspended by
an electrically insulating thread. The three discs are all parallel to each other and their
centers lie along the same horizontal line (so that when viewed head-on the discs are
concentric circles). The resulting set-up is shown in Fig. 1c.
3. Find the radius R of the third disc such that the net electrostatic force acting on each
charged disc is zero. (The fringing effect is neglected in this problem).

Figure 1: Charged discs set-up

You might also like